Explain please!
Hi, This question was really confusing for me. Will someone walk me through it? Thanks!
hkolon on May 29, 2020
  • June 2013 LSAT
  • SEC1
  • Q16
1
Reply
Why is E wrong?
Hello, I chose E and I cannot see why it is wrong. Doesn't the passage say that while profess...
hkolon on May 29, 2020
  • June 2013 LSAT
  • SEC1
  • Q21
1
Reply
A vs C
Hi! Could someone please elaborate on why answer choice A was wrong, and answer choice C was corr...
DavidW on May 11, 2020
  • June 2013 LSAT
  • SEC1
  • Q5
2
Replies
Why could C not be the answer here?
I was struck between D and C on this question but I though C was the better answer.
MACZ on April 23, 2020
  • June 2013 LSAT
  • SEC1
  • Q3
2
Replies
Mapping
I tried to map this question and couldn’t combine any of the premises to find a conclusion. Is th...
shafieiava on April 14, 2020
  • June 2013 LSAT
  • SEC1
  • Q21
1
Reply
PT 69 LR1 Q18
Hi, can someone explain this question to me and why each answer is incorrect/correct? Thank you!
mfujii on February 4, 2020
  • June 2013 LSAT
  • SEC1
  • Q18
1
Reply
Is the reason that A is wrong because we don't ...
I see why E is correct but I was just wanting some further clarification on A.
MACZ on January 13, 2020
  • June 2013 LSAT
  • SEC1
  • Q12
1
Reply
Couldn't figure out the correct answer between ...
I can see how both A and C could work, but didn't quite understand how to choose between these two.
samxinghaoli on December 27, 2019
  • June 2013 LSAT
  • SEC1
  • Q23
1
Reply
Why not B?
Can you explain why B is not the correct answer?
nivensdc on December 27, 2019
  • June 2013 LSAT
  • SEC1
  • Q15
1
Reply
Help
Can you please diagram the statements?
vijetakanabar90 on November 12, 2019
  • June 2013 LSAT
  • SEC1
  • Q7
1
Reply
Question
Why is the answer A and not D?
Zariyah-Hodge on October 24, 2019
  • June 2013 LSAT
  • SEC1
  • Q2
1
Reply
Why is B Incorrect?
Could someone please explain why answer choice B is incorrect; and how this sentence from the sti...
SarahA on July 2, 2019
  • June 2013 LSAT
  • SEC1
  • Q21
4
Replies
Please explain
What is the rational for this ?
Cristianyuma@hotmail.com on November 7, 2018
  • June 2013 LSAT
  • SEC1
  • Q13
1
Reply
Unfinished answer
The right answer is incomplete.
coachz on June 5, 2018
  • June 2013 LSAT
  • SEC1
  • Q1
1
Reply
What is the flaw?
I identified the correct answer because it matches the reasoning in the stimulus, but I'm having ...
Kyle on May 29, 2016
  • June 2013 LSAT
  • SEC1
  • Q14
1
Reply